LSAT and Law School Admissions Forum

Get expert LSAT preparation and law school admissions advice from PowerScore Test Preparation.

User avatar
 Dave Killoran
PowerScore Staff
  • PowerScore Staff
  • Posts: 5852
  • Joined: Mar 25, 2011
|
#82638
Complete Question Explanation

The correct answer choice is (C).

J’s can only be assigned stall 3 under Template #1. In that template, F or G can be assigned to stall 1, and so answer choice (C) is correct.
 rawatnaman
  • Posts: 1
  • Joined: Aug 28, 2016
|
#28117
Why is G the correct answer ? If G is 1, then F could be second and M could be 5th. Which would make answer choices A and E as also correct. Can anyone please help me out?

Thanks,

Naman
 Shannon Parker
PowerScore Staff
  • PowerScore Staff
  • Posts: 147
  • Joined: Jun 08, 2016
|
#28146
Naman,

Ok so starting with the setup and rules: we have six variables (four lions and two tigers). (Fl, Gl, Hl, Jl)4 & (Kt, Mt)2. We have six stalls with three pairs facing each other. We have the rules: 1. that the tigers stalls cannot be facing each other t~t. 2. A Lion must be in stall one 1=l. 3. H must be in stall 6. 4. Jl must be in a stall numbered one higher than Kt (Jl=Kt+1). 5. K cannot face H.

Drawing some inferences we can see from rule 2 that a lion must be in stall one, rule 3 that it cannot be H, and rule 4 that it cannot be J since J must come directly after K. This leaves us with the split possibility of either F or G being in stall 1.

From Rule 4, we know that K cannot be in stall 5 because J would have to be in stall 6 and per Rule 3, H is already there. Since rule 5 states that H and K cannot face each other this means that K cannot be in stall 3. The setup leaves us with a diagram looking something like this.

_...............Hl
3...............6
~Kt

_............... _
2............... 5
.............~Kt

F/Gl..........._
1..............4

The question is a local question which adds the condition that J is in stall 3. If we place J in stall 3 then K must be in stall 2. Since Kt, and Mt cannot face each other (Rule 1) then Mt must go in stall 4. This leave Fl, and Gl to fill slots 1 and 5. Since Klmust be in stall 2, Flcannot be and therefore (A) is incorrect. Since Mt must be in stall 4, it cannot be in stall 5, and therefore (E) is incorrect.

Jl............... Hl
3.............6

Kt............... _
2...............5

Fl/Gl.......... Mt
1.............4


I hope this helps.

~Shannon

Get the most out of your LSAT Prep Plus subscription.

Analyze and track your performance with our Testing and Analytics Package.